Difference between revisions of "2004 AMC 12A Problems/Problem 16"

(Problem)
m
Line 1: Line 1:
 
== Problem ==
 
== Problem ==
The [[set]] of all [[real number]]s <math>x</math> for which
+
The set of all real numbers <math>x</math> for which
  
 
<cmath>\log_{2004}(\log_{2003}(\log_{2002}(\log_{2001}{x})))</cmath>
 
<cmath>\log_{2004}(\log_{2003}(\log_{2002}(\log_{2001}{x})))</cmath>
Line 6: Line 6:
 
is defined is <math>\{x|x > c\}</math>. What is the value of <math>c</math>?
 
is defined is <math>\{x|x > c\}</math>. What is the value of <math>c</math>?
  
<math>\text {(A)} 0\qquad \text {(B)}2001^{2002} \qquad \text {(C)}2002^{2003} \qquad \text {(D)}2003^{2004} \qquad \text {(E)}2001^{2002^{2003}}</math>
+
<math>\text {(A) } 0\qquad \text {(B) }2001^{2002} \qquad \text {(C) }2002^{2003} \qquad \text {(D) }2003^{2004} \qquad \text {(E) }2001^{2002^{2003}}</math>
  
 
== Solution ==
 
== Solution ==
We know that the domain of <math>\log_k n</math>, where <math>k</math> is a [[constant]], is <math>n > 0</math>. So <math>\log_{2003}(\log_{2002}(\log_{2001}{x})) > 0</math>. By the definition of [[logarithm]]s, we then have <math>\log_{2002}(\log_{2001}{x})) > 2003^0 = 1</math>. Then <math>\log_{2001}{x} > 2002^1 = 2002</math> and <math>x > 2001^{2002}\ \mathrm{(B)}</math>.
+
For all real numbers <math>b</math> such that <math>b>0</math> and <math>b\neq1,</math> note that:
 +
<ol style="margin-left: 1.5em;">
 +
  <li><math>\log_b a</math> is defined if and only if <math>a>0.</math></li><p>
 +
  <li><math>\log_b a>c</math> if and only if <math>a>b^c.</math></li><p>
 +
</ol>
 +
Therefore, we have
 +
<cmath>\begin{align*}
 +
\log_{2004}(\log_{2003}(\log_{2002}(\log_{2001}{x}))) \text{ is defined} &\implies \log_{2003}(\log_{2002}(\log_{2001}{x}))>0 \\
 +
&\implies \log_{2002}(\log_{2001}{x})>1 \\
 +
&\implies \log_{2001}{x}>2002 \\
 +
&\implies \boxed{\text {(B) }2001^{2002}}.
 +
\end{align*}</cmath>
 +
~Azjps (Fundamental Logic)
 +
 
 +
~MRENTHUSIASM (Reconstruction)
  
 
== See also ==
 
== See also ==

Revision as of 02:50, 10 July 2021

Problem

The set of all real numbers $x$ for which

\[\log_{2004}(\log_{2003}(\log_{2002}(\log_{2001}{x})))\]

is defined is $\{x|x > c\}$. What is the value of $c$?

$\text {(A) } 0\qquad \text {(B) }2001^{2002} \qquad \text {(C) }2002^{2003} \qquad \text {(D) }2003^{2004} \qquad \text {(E) }2001^{2002^{2003}}$

Solution

For all real numbers $b$ such that $b>0$ and $b\neq1,$ note that:

  1. $\log_b a$ is defined if and only if $a>0.$
  2. $\log_b a>c$ if and only if $a>b^c.$

Therefore, we have \begin{align*} \log_{2004}(\log_{2003}(\log_{2002}(\log_{2001}{x}))) \text{ is defined} &\implies \log_{2003}(\log_{2002}(\log_{2001}{x}))>0 \\ &\implies \log_{2002}(\log_{2001}{x})>1 \\ &\implies \log_{2001}{x}>2002 \\ &\implies \boxed{\text {(B) }2001^{2002}}. \end{align*} ~Azjps (Fundamental Logic)

~MRENTHUSIASM (Reconstruction)

See also

2004 AMC 12A (ProblemsAnswer KeyResources)
Preceded by
Problem 15
Followed by
Problem 17
1 2 3 4 5 6 7 8 9 10 11 12 13 14 15 16 17 18 19 20 21 22 23 24 25
All AMC 12 Problems and Solutions